Representaciones irreducibles unitarias del pequeño grupo SO(3)SO(3)SO(3)

Actualmente estoy trabajando con el método de representaciones inducidas para calcular las representaciones unitarias irreducibles del grupo de Poincaré.

Convenciones/Notación

Firma métrica = ( , + , + , + )

C = = 1

Los índices i,j,k superan 1,2,3 mientras que cualquier otro índice latino supera 0,1,2,3.

X a = ( t , X )

Contexto

En este hilo consideraré el caso de masa distinta de cero (definida positiva). Tomaré el impulso estándar como k a = ( metro , 0 , 0 , 0 ) donde m es la raíz cuadrada del valor propio del operador de Casimir C 1 = PAG a PAG a . El grupito correspondiente es H k = S O ( 3 ) .

El segundo operador casimiro es el cuadrado del vector de Pauli-Lubanski; C 2 = W a W a . Para el momento estándar, W a = ( 0 , metro j i ) dónde j i es el i-ésimo generador de rotaciones (es decir, alrededor del i-ésimo eje espacial). De este modo, C 2 = metro 2 j j . De QM conocemos los valores propios de j 2 son s ( s + 1 ) . Entonces, por el lema de Schur, en cualquier irrep del grupo de Poincaré, C 2 = metro 2 s ( s + 1 ) Identificación . Id es el operador de identidad... \mathbb{1} no funcionará.

Por lo tanto, si fijo un valor para m (y por lo tanto fijo la superficie de la capa de masa), entonces las irrepeticiones unitarias del grupo de Poincaré (para esta m) se clasifican por el 'espín', s.

En este caso, el método de repeticiones inducidas se puede expresar como:

tu ( Λ ) | pag , σ = Σ σ D ( s ) ( h ( Λ , pag ) ) σ σ | Λ pag , σ
Dónde h ( Λ , pag ) = L 1 ( Λ pag ) Λ L ( pag ) H k y L ( pag ) es la transformación estándar de Lorentz que lleva el impulso estándar a p, es decir L ( pag )     b a k b = pag a . También, D ( s ) ( h ( Λ , σ ) ) es la irrep unitaria del elemento del pequeño grupo h ( Λ , σ ) en la representación del espín.

Así que para averiguar cómo la transformación homogénea de Lorentz tu ( Λ ) actúa sobre el estado | pag , σ , necesitamos resolver los irreps unitarios del pequeño grupo.

Por convención tomaré la transformación estándar de Lorentz, L ( pag ) , ser

L ( pag )       b a = ( mi pag / metro pag j / metro pag i / metro d i j + pag i pag j metro ( mi pag + metro ) ) ,                       i , j = 1 , 2 , 3.

Pregunta/Intento de resolver

Desafortunadamente, en mi clase de QM no relativista durante la licenciatura, nunca hablamos sobre las representaciones de espín del grupo de rotación. S O ( 3 ) , así que estoy tratando de cerrar esa brecha ahora. Estoy un poco confundido sobre cómo vamos a construir los irreps unitarios de S O ( 3 ) , en particular, no estoy tan seguro de cómo calcular los elementos de la matriz D ( s ) ( h ( Λ , σ ) ) σ σ . Esta es mi pregunta, y lo siguiente es lo que tengo hasta ahora.

Al tratar de traducir entre lo que aprendí en QM de pregrado y lo anterior, llegué a las siguientes conclusiones:

  • Para el caso masivo (y para el momento estándar de modo que estemos en el marco de reposo), el σ 's (que se supone que representan cualquier grado de libertad que no sea 4-momentum) corresponden al 'número cuántico magnético', metro j .
  • Para un giro dado s, σ = metro j = { s , s + 1 , . . , s 1 , s } . aquí estoy tomando j = yo + s = s ya que estamos en el marco de reposo y no hay 'momento angular orbital' ( yo = 0 ). Por tanto, para una masa dada, m, la dimensión de la irrep del pequeño grupo es 2 s + 1 .

Definir el 3-vector S i = 1 metro W i = 1 2 ε i j k j j k S S = s ( s + 1 ) Identificación que obedece al siguiente álgebra:

[ S i , S j ] = i ε i j k S k
De este modo [ S i , S 2 ] = [ 1 2 ε i j k j j k , 1 metro 2 C 2 ] = 0 (desde C 2 es un operador Casimir de ISO ( 3 , 1 ) ). Por lo tanto, podemos diagonalizar simultáneamente S 2 y uno de S i , decir, S 3 .

De la forma habitual podemos calcular que:

S 2 | s , σ = s ( s + 1 ) | s , σ                                                               ( 1 )
S 3 | s , σ = σ | s , σ                                 ( dónde  σ = metro s )       ( 2 )
Y defina operadores de subida/bajada:
S ± = S 1 ± i S 2
S ± | s , σ = s ( s + 1 ) σ ( σ ± 1 )   | s , σ ± 1         ( 3 )

Según Weinberg, para una rotación infinitesimal, R i k = d i k + Θ i k , tenemos eso

D ( s ) ( 1 + Θ ) σ σ = d σ σ + i 2 Θ i k ( j ( s )   i k ) σ σ

Para obtener una representación de una rotación finita necesitamos exponenciar el caso infinitesimal:

D ( s ) ( R ) = Exp { i Θ i k j i k }
Lo cual, creo, se traduce en mi notación a:
D ( s ) ( R ( θ ) ) = Exp { i θ S }
Para una rotación de un ángulo | θ | alrededor θ , junto con la observación de que S i = 1 2 ε i j k j j k S = ( j 23 , j 31 , j 12 ) = ( j 1 , j 2 , j 3 ) = j .

Entonces podemos calcular las representaciones unitarias del pequeño grupo S O ( 3 ) por:

D ( s ) ( R ( θ ) ) σ σ = s , σ | mi i θ S | s , σ                         ( 4 )

Podemos calcular esto explícitamente usando las ecuaciones (2) y (3), y así para un s fijo, el tamaño de la matriz D será (2s+1)(2s+1).

Creo que lo que he dicho hasta ahora es mayormente correcto (pero me gustaría alguna confirmación). Si lo que he dicho es correcto, entonces el problema que tengo es que no sé traducir entre D ( s ) ( h ( Λ , pag ) ) σ σ y D ( s ) ( R ( θ ) ) σ σ . Por ejemplo, dado (4), ¿cómo puedo calcular los elementos de matriz correspondientes de D ( s ) ( h ( Λ , pag ) ) σ σ ? Sé que debe haber una relación porque ambos son solo rotaciones. Entonces puedo inferir que θ = θ ( Λ , pag ) . Pero parece que la forma estándar de calcular los elementos de matriz deseados es usar las transformaciones estándar de Lorentz que proporcioné en la sección de contexto... pero a mí me parece que esto solo es útil si tenemos un método explícito para calcular los elementos. de la representación de h ( Λ , pag ) , como la ecuación (4). ¿Existe una expresión tan explícita? Salud.

1. Creo que lo que has escrito es correcto. 2. No estoy seguro de cuál es la pregunta real. Creo que la mejor manera de calcular estos elementos de matriz sería calcular θ ( Λ , pag ) . Hacer esto parece tedioso pero factible: puede calcular explícitamente L 1 ( Λ pag ) Λ L ( pag ) , obtener una matriz de rotación y "leer" θ , ¿bien?

Respuestas (2)

Esta respuesta se basa en este artículo de A. Ungar.

Ungar calculó la fórmula de rotación de Thomas, que es casi lo que necesita. Describiré el procedimiento general y, en algunos casos, lo remitiré a Ungar para obtener la prueba. Expresaré (al igual que Ungar), los impulsos en términos de velocidades en lugar de momentos. Si lo desea, puede repetir el ejercicio con la parametrización del impulso. De Wikipedia tenemos

B ( v ) = [ γ v γ v C v t γ v C v 1 ( 3 × 3 ) + γ v 2 1 + γ v v v t C 2 ]

El punto clave para encontrar la rotación de Wigner es la observación de que cada transformación de Lorentz se puede descomponer (de manera no única) como un producto de un impulso y una rotación:

Λ = B ( tu ) R
Cualquier elección del método de descomposición servirá, pero necesitamos trabajar en un método fijo de descomposición. Te describiré un posible método al final de la respuesta. Ahora, cuando multiplicamos dos impulsos, obtenemos un impulso con la velocidad de adición relativista + una rotación (a menudo denominada rotación de Thomas):
B ( tu ) B ( v ) = B ( tu v ) T o metro ( tu , v )
Dónde T o metro es una matriz de rotación Ungar encontró la solución general para la rotación de Thomas (Ecuación: (13) en el artículo)
T o metro ( tu , v ) = B ( tu v ) B ( tu ) B ( v )
Ahora, estamos en condiciones de resolver la ecuación de la rotación de Wigner. Necesitamos resolver:
Λ B ( v ) = B ( Λ v ) W
para W . Parametrizamos Λ , obtenemos para el lado izquierdo:

Λ B ( v ) = B ( tu ) R B ( v ) = B ( tu ) R B ( v ) R 1 R = B ( tu ) B ( R v ) R = B ( tu R v ) T o metro ( tu , R v ) R

y para el lado derecho

B ( Λ v ) W = B ( B ( tu ) R v ) W = B ( tu R v ) W

De este modo:

W = T o metro ( tu , R v ) R
Lo que queda es describir una parametrización específica de una matriz general de Lorentz en un impulso y una rotación: Necesitamos encontrar B ( tu ) y R tal que:
[ λ 0 ξ t η Λ 1 ] = [ γ v γ v C v t γ v C v 1 ( 3 × 3 ) + γ v 2 1 + γ v v v t C 2 ] [ 1 0 0 R 1 ]

observamos que R 1 necesita satisfacer la siguiente relación

ξ = R 1 η
De este modo R 1 necesita rotar el 3-vector η en el 3-vector ξ La solución (usando un método similar a Ungar) se puede escribir como:
R 1 = 1 ( 3 × 3 ) + pecado ( θ ) Ω + ( porque ( θ 1 ) ) Ω 2

Dónde θ es el ángulo entre los vectores η y ξ

pecado ( θ ) = η × ξ | η | | ξ |
y
Ω i j = η i ξ j ξ i η j | η | | ξ |

Gracias por la respuesta. El enlace al artículo al que hace referencia no funciona, ¿podría corregir el enlace o escribir el nombre del artículo y la revista en la que se encuentra? Además, ¿crees que este método podría emplearse para responder la segunda pregunta que tengo en la publicación que figura a continuación? física.stackexchange.com/questions/349836/…
Lo siento, lo intenté dos veces y cada vez que el enlace funciona inicialmente, luego deja de funcionar. De todos modos, este es el enlace de una página anterior en Google Scholar, que también pondré en el texto. erudito.google.co.il/…

Para complementar la respuesta anterior, las matrices de espín para espín arbitrario tienen expresiones explícitas compactas [1][2].

( S X ) a b = 2 ( d a , b + 1 + d a + 1 , b ) ( s + 1 ) ( a + b 1 ) a b ( S y ) a b = i 2 ( d a , b + 1 d a + 1 , b ) ( s + 1 ) ( a + b 1 ) a b 1 a , b 2 s + 1 ( S z ) a b = ( s + 1 a ) d a , b = ( s + 1 b ) d a , b .

Las fuentes contienen expresiones explícitas para girar 1 , 3 2 , 2 , y 5 2 . Con estos puedes exponenciar para obtener la representación explícita de cualquier rotación en cualquier spin- j objeto:

D σ σ ( j ) ( R ( θ ) ) = D σ σ ( j ) ( mi i θ S )

En general, el resultado para espín arbitrario j no es tan bonito, como es el caso de spin- 1 / 2 (y posiblemente girar- 1 ).

Para resumir, con los resultados indicados por David Bar Mosche (arriba), puede calcular el vector de rotación de Thomas θ correspondiente a la rotación de Wigner. Luego, con estos resultados, puede calcular explícitamente la acción de esa rotación de Thomas (bueno, cualquier rotación espacial arbitraria en realidad) en un spin- j objeto para cualquier j .


[1] https://en.wikipedia.org/wiki/Spin_(física)#Higher_spins

[2] https://arxiv.org/pdf/1402.3541.pdf